HN Theater @HNTheaterMonth

The best talks and videos of Hacker News.

Hacker News Comments on
Bell's Theorem: The Quantum Venn Diagram Paradox

minutephysics · Youtube · 25 HN points · 23 HN comments
HN Theater has aggregated all Hacker News stories and comments that mention minutephysics's video "Bell's Theorem: The Quantum Venn Diagram Paradox".
Youtube Summary
Featuring 3Blue1Brown
Watch the 2nd video on 3Blue1Brown here: https://www.youtube.com/watch?v=MzRCDLre1b4

Support MinutePhysics on Patreon! http://www.patreon.com/minutephysics
Link to Patreon Supporters: http://www.minutephysics.com/supporters/

This video is about Bell's Theorem, one of the most fascinating results in 20th century physics. Even though Albert Einstein (together with collaborators in the EPR Paradox paper) wanted to show that quantum mechanics must be incomplete because it was nonlocal (he didn't like "spooky action at a distance"), John Bell managed to prove that any local real hidden variable theory would have to satisfy certain simple statistical properties that quantum mechanical experiments (and the theory that describes them) violate. Since then, GHZ and others have managed to extend the theoretical work, and Alain Aspect performed the first Bell test experiment in the late 1980s.

Thanks to Vince Rubinetti for the music: https://soundcloud.com/vincerubinetti/one-two-zeta

And thanks to Evan Miyazono, Aatish Bhatia, and Jasper Palfree for discussions and camaraderie during some of the inception of this video.

REFERENCES:

John Bell's Original Paper: http://inspirehep.net/record/31657/files/vol1p195-200_001.pdf

Quantum Theory and Reality: https://www.scientificamerican.com/media/pdf/197911_0158.pdf

"What Bell Did" By Tim Maudlin: https://arxiv.org/pdf/1408.1826

Bell's Theorem on Wikipedia: https://en.wikipedia.org/wiki/Bell%27s_theorem

2015 experimental confirmation that QM violates Bell's theorem: https://arxiv.org/pdf/1508.05949.pdf
https://journals.aps.org/prl/abstract/10.1103/PhysRevLett.115.250402

Bell's Theorem without Inequalities (GHZ): http://dx.doi.org/10.1119/1.16243

Kochen-Specker Theorem: https://en.wikipedia.org/wiki/Kochen–Specker_theorem

MinutePhysics is on twitter - @minutephysics
And facebook - http://facebook.com/minutephysics
And Google+ (does anyone use this any more?) - http://bit.ly/qzEwc6

Minute Physics provides an energetic and entertaining view of old and new problems in physics -- all in a minute!

Created by Henry Reich
HN Theater Rankings

Hacker News Stories and Comments

All the comments and stories posted to Hacker News that reference this video.
What you are describing is a "hidden variable" theory. They are disproved by the experiments of the Bell's inequality. It's more weird, much more weird.

Let's continue with your experiment about the pair of red or green socks. If you and your friend measure if they are red-or-green, both will get the same results. This can be explained with a classical theory. Nobody disagree with that.

The weird part is that you can measure if they are 50%red and 50%green! Can I call it yellow? This makes no sense with classical socks and colors, but it makes sense for quantum particles and other properties.

But there are two ways to combine 50%red and 50%green, the technical notation is (R+G)/sqrt(2) and (R-G)/sqrt(2), one with a plus and one with a minus. Can I call them good-yellow and bad-yellow? Or you prefer yellow and blue? In one of the experiments, red means vertical and green horizontal, so one of the combinations is a 45° diagonal like this / and the other is a 45° diagonal like this \. You don't need fancy equipment to measure the combinations, it's just a polarizer rotated 45°. Can I call them yellow and backyelow? I prefer good-yellow and bad-yellow because it's more clear that something weird is happening.

If you measure red-or-green and your friends measures good-yellow-or-bad-yellow, then the results will not be correlated. If you got "red", your friend has a 50% probability of getting good-yellow and a 50% probability of getting bad-yellow. There is nothing to explain here.

If you and your friend measure if they are good-yello or bad-yellow, both will get again the same results. This can be again explained with a hidden variable theory. Both socks "know" what to say if they are asked if they are red-or-green and what to say if they are asked if the are good-yellow-or-bad-yellow.

It get's more interesting when you pick more combinations, like 90%red and 10%green. Can I all it orange? And you can pick 10%red and 90%green. Can I call it lemon? If you measure red-or-green and your friends measures orange-or-lemon, then if you got red, your friend will get orange 90% of the time.

And there are good-orange, bad-orange, good-lemmon and bad-lemmon. And there are many more shades of orange-yellow-lemon. But this is getting too long.

You can have very smart socks that know what to answer for every possible combination of colors. So if you and your friend ask for the same color, whatever it is, both get the same result.

The problem is when you and your friend measure many times using the correct shades of orange and lemon. So the results don't agree 0% neither 100%. You can count how many times you get each combination of results, like (red-vs-dark-orange, or green-vs-bright-yellow), and then add and subtract some of them.

If you assume the socks can's communicate with the other socks before answering, then the result of the calculation is smaller then some number. But in the experiments disagree.

There are some videos with all of this, with a better and longer explanation by MinutePhysics and 3Blue1Brown https://www.youtube.com/watch?v=zcqZHYo7ONs and https://www.youtube.com/watch?v=MzRCDLre1b4&t=0s

d0mine
It reminds me boxes with 3 green/red lights from the workshop "Quantum Mechanical View of Reality" by Richard Feynman https://youtu.be/ZcpwnozMh2U?t=18m20s

(there is a statement that 3 is the minimum, 2 lights won't work)

Sep 16, 2022 · 3 points, 1 comments · submitted by Amorymeltzer
injb
I'm trying to understand the basics of quantum mechanics and Bell's Theorem in my spare time. This is a good video but it's still hard to understand.

I'm beginning to think that Bell's Theorem doesn't say that there aren't hidden variables, but just that hidden variables don't fix the problem of non-locality. Thus they are not impossible, but just unnecessary.

So far the only part of QM I've really been able to understand is Leonard Susskind's Theoretical Minimum stuff (video lectures & book) about electron spin.

There are reliable ways to create superposition particles: when you measure in one direction, the other direction is a superposition (https://physics.stackexchange.com/questions/43974/how-can-we...). I have no idea how to create entangled superposition particles: maybe read https://www.eurekalert.org/news-releases/473741.

You can tell the particles are in a superposition because they behave differently than those in a static state:

- Double-split experiment: https://en.wikipedia.org/wiki/Double-slit_experiment - only particles in a superposition create interference patterns

- Bell's paradox: https://www.youtube.com/watch?v=v7jctqKsUMA, https://www.youtube.com/watch?v=hiyKxhETXd8 - entangled particles in a superposition can be measured to make predictions with more confidence than if they were simply static particles with different states

I guess a good search term would be "Polarized light and quantum mechanics" because IMO polarized light is a good example which you can even simulate yourself if you have a polarizing filter (https://www.youtube.com/watch?v=zcqZHYo7ONs&t=736s, https://www.youtube.com/watch?v=ZudziPffS9E, https://chem.libretexts.org/Bookshelves/Physical_and_Theoret...)

ngcc_hk
The polarised one is really crazy as it is macroscopic and daily life.
denton-scratch
It's also dead easy to do - use a cheap laser pointer, a tinfoil cap with a pinhole, and some polarising lenses from an optician (he gave me a couple for free).
Yeah it is a common misconception. There is also a minutephysics video on it (https://youtu.be/zcqZHYo7ONs) that portrays the effect of polarising filters in a misleading/wrong way, leading to further confusion.

Polarisation is used in many books and courses as a familiar classical system, which we then use to springboard into Stern-Gerlach, which actually is an inherently quantum effect.

Polarisation already appears when writing down the full solution to the classical EM wave equation. On the other hand, the similar effect seen in the Stern-Gerlach experiment with the angular momentum measurements is not: the non-commutivity of the angular momentum operator for different measurement axes is a purely quantum phenomenon.

The concern that the article presents - that the middle filter influences the light and thus allows it to pass through the third filter - is actually addressed in popular quantum mechanics explanations that use the 3 filter experiment.

They say that if we use two entangled photons and let them fly far apart, then pass one of them through two filters, and the second photon through the middle filter, the first photon will be affected - it will get a chance to pass though the pair of filters.

That they say is "spooky action at distance" - the second photon will influence behaviour of the first photon at the remote site of the experiment and the "influence" is faster then the speed of light.

Example here by MinutePhysics and 3Blue1Brown: https://youtu.be/zcqZHYo7ONs Explanation about entanglement starts at around 8:50.

But even with that addressed, to me personally this video is not satisfying.

If the spooky action at distance can be observed so trivially - choosing a filter at one site site affects what happens at the remote site - we don't need a mathematical inequality (the Bell's inequality), it's already so obviously spooky.

There are also serious problems with clarity of their explanation, as I commented in https://www.youtube.com/watch?v=zcqZHYo7ONs&lc=Ugz3tzpDP_i1N... and https://www.youtube.com/watch?v=zcqZHYo7ONs&lc=Ugz3tzpDP_i1N...

I am not sure the real Bell experiments are really done using 3 polarizing filters and will the effect really be observed in experiment with two remote sites.

My conclusion, it's problematic to rely on "pupular science" explanations, even by good channels like MinutePhysics and 3Blue1Brown.

abbeyj
> They say that if we use two entangled photons and let them fly far apart, then pass one of them through two filters, and the second photon through the middle filter, the first photon will be affected - it will get a chance to pass though the pair of filters.

I don't think that can be correct. This would allow FTL communication of information. I could put the filter in the path of the second photon when I wanted to send a 1 and leave it out of the path of the second photon when I wanted to send a 0. In the 1 case the other side would sometimes see the photon pass both filters. In the 0 case the other side would never see the photon pass both filters. Combine this with a little error correction and you have a channel for transmitting arbitrary information faster than light which would violate the no-communication theorem. https://en.wikipedia.org/wiki/No-communication_theorem

The MinutePhysics video you link describes a different setup with only one filter in the path of each photon. This gives you some information about the probability that other photon got past its filter but it doesn't let you transmit anything.

avodonosov
Yes, thank you, in distributed setting they really use only two filters.

But my main point is that the video addresses the issue from the article.

The video lays out intuition for the Bell's theorem using the 3 filter experiment. And then, at 8:50 they say: "what if the act of passing through one filter changes how the photon will later interact with other filters? Then you can easily explain the results of the experiment" and continue explaining that the real spookiness can be proven in a distributed experiment.

It's claimed that polarization is due to quantum effects.

https://www.youtube.com/watch?v=zcqZHYo7ONs

…I think it actually isn't and the people explaining it have just forgotten it's supposed to be a metaphor though.

rcxdude
polarization is very much a quantum effect, however the specific experiment in the video does not prove the quantum wierdness because the setup does allow a hidden variable explanation.
Jan 28, 2022 · guerrilla on Why Quantum Mechanics?
> I have a lot of trouble imagining any experiment which could completely rule out every other influencing factor

It's called Bell's theorem and it can even be tested at home[4]. There are no local hidden variables in quantum mechanics.[1][2][3]

1. https://en.wikipedia.org/wiki/Bell%27s_theorem

2. https://en.wikipedia.org/wiki/EPR_paradox

3. https://en.wikipedia.org/wiki/Hidden-variable_theory

4a. https://www.youtube.com/watch?v=zcqZHYo7ONs

4b. https://www.youtube.com/watch?v=MzRCDLre1b4

lulantivaxxers
Thanks for this, hidden variable theories are very seductive but Bell's Theorem is definitive.
naasking
It's definitive that you have to give up locality (Bohmian mechanics), counterfactual definiteness (Copenhagen and others), or statistical independence (Superdeterminism). It's not definitive at all about local hidden variables.
guerrilla
> It's not definitive at all about local hidden variables.

What?

> To date, Bell tests have found that the hypothesis of local hidden variables is inconsistent with the way that physical systems do, in fact, behave.

naasking
Bell's theorem assumes statistical independence in its proof that local hidden variables can't reproduce QM. Superdeterminism violates statistical independence, therefore Bell's theorem does not rule out a superdeterministic local hidden variables theory, like this one:

https://arxiv.org/abs/2010.01327v5

guerrilla
Oh, thank you. I've been meaning to check her blog on this.
Jul 27, 2021 · palijer on Naval Architecture
This is one of those physics phenomenon where I feel like they are a software bug. Bell's Theorem and a lot of quantum entanglement stuff is like that as well.

https://youtu.be/zcqZHYo7ONs

jschwartzi
It’s actually quite intuitive, as the force is distributed over a larger area. So although the pressure gradient isn’t affected by the discontinuity in the container size, if you compare forces exerted by the pressure on a plate in either section of the chamber you’ll observe that the force on the wider plate would be reduced to compensate for the increased area in the presence of the same pressure.
Ma8ee
The force would scale with the area, since pressure is force per area. Not the other way around.
garmaine
I'm not sure why you're being down-voted. If you double the size of a water column, you of course double the total weight pressing down. But you've also doubled the cross-sectional area, so the weight-per-unit-area (pressure) remains the same. This is pretty intuitive if you understand what pressure is.
dan-robertson
Perhaps because the explanation (and indeed concept) isn’t that intuitive so the comment may be read as dismissive?

If you see pressure as coming from the weight of the fluid above and now replace fluids with solids for a mental model, it obviously doesn’t work: imagine two parallel rigid plates connected by some rigid structure. The top plate will represent the (fluid) boundary between the top of the barrel and the tube and the bottom plate the bottom of the barrel. If you put a narrow column of metal on the top of the top plate, then the pressure exerted by the bottom plate is, say, p. If you make that column much wider, increasing the weight above, the pressure is say 10p.

I think the problem is that this intuitive model of pressure is just wrong but if it doesn’t come from the weight of the water above, it is hard to intuitively see where it does come from.

garmaine
Your example is completely different though. Considering neighboring water columns isn't like adding more to the top of the plate, it's like setting up an entirely different plate next door. Which of course doesn't affect the first instance at all.
dan-robertson
The point is that the way people typically think of the problem leads to examples like mine, and this is why the concept is unintuitive. It doesn’t matter whether the model is correct—the whole point is that it is the incorrect model many people will have or start with.
I can also suggest a video that seemed to me to explain Bell's theorem and what it means that the correlations are stronger than classically possible here:

[0] https://www.youtube.com/watch?v=zcqZHYo7ONs

Part of this is because of tests like the ones involved with Bells theorem. Minute physics does a better job explaining it then I can. https://m.youtube.com/watch?v=zcqZHYo7ONs&vl=en
You can simulate quantum effects with just polarized glasses.

https://www.youtube.com/watch?v=zcqZHYo7ONs

ASalazarMX
Oil droplets can model quantum behavior too https://www.youtube.com/watch?v=WIyTZDHuarQ
andi999
You can't. This is just covered by classical electrodynamics. Of course in a sense you can say anything is quantum, but then the stability of a table is quantum (which it is). If you were to have a single photon source and avalanche detector, ok now you are talking quantum.
admax88q
I'm curious, can you detail how classic electrodynamics covers this?
kgwgk
https://aip.scitation.org/doi/pdf/10.1063/1.4943751
infogulch
The first 2 minutes of that video make it clear that they think it is "real" quantum effects (e.g. at 1:34 entanglement > EPR paradox > Bell's Theorem > local realism). Those are pretty quantum-y imo.
andi999
If you have entanglement then it is quantum. Yes. But the comment was you 'just' need polarized glasses. So if 'just' polarized glasses for a few $ means: "and of course a biphoton source for a couple of thousand $ and some coincidence counters for another couple of thousand $", then you only 'just' need polarizers.
tofof
Yeah. At 9:34 they're dealing with entangled photons passing through 2 filters at different points in space, simultaneously. They affect each other, which is impossible without faster-than-light communication between them. This is absolutely quantum and cannot be explained at all by classical EM.
Interestingly the page for polarization of light has a very significant mistake in it. Since the demo allows the use of more than two polarizing filters, what is missing is the quantum effect that occurs when you have 3. When the outer two are the same polarization, and the middle one is opposing, the light will show through at approximately 50% even though it seems to be "stopped" at the second lens.

It's not the same as the double-slit example, but is related to it.

Either way, this site is really good. Unfortunately the first thing I tried to replicate didn't work because they haven't taken quantum effects into account.

Here is a video about the effect with 3 polarized lenses - it sounds crazy enough I want to add a bit of proof to avoid downvotes: https://www.youtube.com/watch?v=zcqZHYo7ONs

And here is Action Labs demonstrating something related, where light seemingly shines through a solid ball for a slightly different, but not unrelated reason: https://www.youtube.com/watch?v=TM9alPcOMcU

Feb 08, 2021 · 1 points, 0 comments · submitted by fagnerbrack
[0] is a 20min video which you can replicate for $20 at a convenience store. [1] has Conway explaining in detail, although it's not excruciating. I don't see why the burden of proof is on me; Bell's inequalities are not new.

[0] https://www.youtube.com/watch?v=zcqZHYo7ONs

[1] https://plus.maths.org/content/john-conway-discovering-free-...

AnimalMuppet
The question is whether Bell's inequalities really imply that there is an objective reality. If I understand correctly, the loss of objective reality is one explanation for Bell's inequality, but not the only one.

And no, I'm not going to watch a 20 minute video to find out what your argument is.

And as for the second point, even if you don't like "objective reality" in there, do you deny that there are natural laws that govern things? (Bell's inequality would be one such law.)

Kednicma
My argument is the same one that Bell, Kochen, Specker, Conway, and so many others have covered before: If photons were locally real, then we'd see different measurements from what we actually do see. The video covers the basic idea of witnessing this using polarizing filters, as well as explaining Bell's inequalities using basic discrete set theory and Venn diagrams.

We don't have to lose reality; we could instead lose locality. However, then we also lose most of physical causality and spatial relevance. [0] covers all of the material but takes more than 20min to read. At the end of it, we're left with a very weak theory that has trouble ruling out FTL influences.

On the second point, it doesn't particularly matter whether those natural laws exist; my point was that they're not fully knowable. There's no reason why the universe ought to admit a short legible encoding of its own structure, especially considering that such structure ought not be observable by creatures like us residing in the universe.

Rather than knowing the natural laws, we know predictive models which are good approximations. There is fundamental uncertainty preventing our approximations from sharpening to some arbitrarily fine precision.

[0] https://en.wikipedia.org/wiki/Quantum_nonlocality

AnimalMuppet
Yeah, losing locality instead of reality was what I was referring to. Although that would be pretty weird too.

But losing one or the other, and not knowing which one, is really uncomfortable. That leaves us with your comment about quantum disproving the first two points... well, I can say that it's not proven yet, but I also can't say that you're wrong.

ncmncm
I have long since given up on locality, and so have come to resent the persistent need, every time I move, to force my stubborn body to traverse every Planck length between here and there.

The apparent knowability of the universe, until recently, has always been its most astonishing feature. We should not be surprised to find limits on that; nor that we have not yet actually run up against those limits.

voxl
Superdeterminism.

Also, love the downvotes when you have failed to prove what you set out to. Bell's Inequalities do not rule out a fully determined world.

Kednicma
But Conway's Free Will Theorem certainly does! The superdeterminism cannot help, either; either the world is so superdetermined that falsifiability and the scientific method are totally meaningless, or the world is not sufficiently deterministic and the Free Will Theorem neatly cleans up the remainder by invoking Kochen-Specker.

I recommend that you examine whatever underlying beliefs are forcing you to require determinism! They probably need to be questioned.

This is incorrect [edit: it isn't] - see the standard "three polarizing filter" experiment [0] which is impossible to explain with classical mechanics [edit: it can actually be explained, see coolgod's comment below]. Polarizing filters don't just zero out the perpendicular component of an electric wave, they measure each individual photon that passes through and either blocks it or permits it to pass with some probability proportional to the angle between the photon's polarization and the filter's polarization.

[0] https://youtu.be/zcqZHYo7ONs

radioactivist
The article/your video make the assumption that light is particle and that those particles are independent and each carry polarization. While we know those assumptions to be true, the classical theory of light is not a theory of particles, but of fields/waves. So when you're asking whether there is a classical explanation that's where you should look. And if you do so, you get something perfectly consistent.

[I.e. if you assumed quantum mechanics didn't exist and that Maxwell's equations were the ground truth, you could explain this behaviour without any issue (with some leeway to define a polarizer)].

I think if you try and use that line of thought strictly (only call an explanation "classical" if you can explain with discrete photon particles), you'd probably have to argue that basically all of electromagnetism is fundamentally quantum mechanical. Again, while not strictly wrong (given our present knowledge), this goes too far for me (and I'd imagine most people).

Also: The "three-polarizer" experiment from your quoted video has a perfectly simple explanation in terms of electromagnetic waves. You use a polarizer to get light polarized along say "y" == (0,1). If you put another polarizer in front of it along "x" == (1,0) then the projection is of the E-field is zero and no light passes through. Now add another polarizer at 45 deg between the two: it then projects the E-field onto its axis, mapping it from (0,E) to (E/2,E/2) (magnitude is 1/sqrt(2)). The E-field now has a non-zero component along "x". So light comes out the final polarizer.

justinpombrio
Here's how to do the "three polarizing filter" experiment. You can do it at home if you have three pairs of polarized sunglasses.

1. Take two of the lenses, hold them an inch apart, and shine a light so that it goes through both. The amount of light that goes through depends on their relative angle; at the right angle (90 degrees difference), no light will pass through. Hold them like this, so that no light gets through.

2. Insert a third polarizing filter between the two at a 45 degree angle. Amazingly, some light will now get through. You added an obstacle, and more light passes through.

jes
Thank you for an interesting experiment to try. I’m looking forward to doing it myself.

I want to ask, though. Is it correct to call the third filter an obstacle? In the quantum realm, it’s not really an obstacle, is it?

justinpombrio
> Is it correct to call the third filter an obstacle? In the quantum realm, it’s not really an obstacle, is it?

Evidently not!

coolgod
The three polarizing filter experiment using common sources of light can be explained perfectly without using quantum mechanics [0]. These effects only need QM to explain in the single photon regime. The polarized light from the LED display is definitely not in the single photon regime thus this experiment does not demonstrate any quantum effects. Without any quantum effects, it is much more difficult to justify the quantumness of this supposed quantum computer.

[0] http://alienryderflex.com/polarizer/

ahelwer
I was going to say, this wouldn't explain it because the electric field strength is projected to the polarization axis (with strength 0.707 at 45 degrees) but to match quantum measurement it should be 0.5 strength (0.707 squared). But as the grandparent comment stresses, light intensity is the square of electric field strength... so the measurement matches. Makes sense! I will amend my above comment.
I found these two videos very helpful in understanding the quantum nature of light after being stuck in the same spot: https://www.youtube.com/watch?v=zcqZHYo7ONs https://www.youtube.com/watch?v=MzRCDLre1b4 (Watch those in order, because they're a collaboration between two YouTubers)
Minute Physics and 3brown1blue did a two part collab that simplifies some of it down to a counting problem that'd be suitable for grade school.

https://www.youtube.com/watch?v=zcqZHYo7ONs

https://www.youtube.com/watch?v=MzRCDLre1b4

lpellis
I have seen the example with the polarized lenses in a few places, but they dont explain why (imo) the simplest explanation does not apply. Namely that the lens itself might disturb the phase of the light, which would then mean it can pass through the next lens.
wwarner
The idea is that polarization is only one of many places where the effect is observed.
sooheon
But if you take away the third lens, there is no light of any polarization. How is it that by adding a filter, you create light where there was none?
lpellis
Only if you take away the middle lens, not the third.

Here's what I would have thought happens: After the first lens, you get polarized light, 90deg offset from the last lens, so no light passes. Then you introduce a 3rd lens in the middle, 45deg offset. This could alter the polarization (maybe it widens the band, or introduce some greater variance, shifts it who knows), and this is why now some light will pass through number 3. No need to create any light

sooheon
If it is true that placing the 45 degree lens third or first does not show the same effect, it is much less astonishing.
Why Quantum Mechanics I have no idea.

Why interpretations: There is an experiment you can do that is hard to explain: Either particles are able to somehow influence each other faster than light (non-local), or the particle somehow doesn't exist except when interacting with some other particle (non-real).

Try this video: https://www.youtube.com/watch?v=zcqZHYo7ONs the AHA moment in the video comes when you realize you can entangle the light and that adding a filter by one stream of light somehow causes the other stream of light to also be influenced.

My goal wasn't to explain a "correct" view quantum physics, only to demonstrate at a cursory level that a binary computer's bits differs from a quantum computer's qbits. Quantum physics is not intuitive and attempting to use a classical understanding of probability will get you in trouble. I highly suggest looking at videos on YouTube, which have a far better explanations on the actual behavior of quantum computation. This is a very good video on Bell's Inequality: https://www.youtube.com/watch?v=zcqZHYo7ONs&t=856s
Oct 24, 2019 · 3 points, 0 comments · submitted by wwarner
A bell inequality is a feature of quantum systems by which they break our understanding of classical probability. A good reference is here [1]. The basic idea is to imagine a Venn diagram where the sum of the parts adds up to more than 100%. Bell inequalities are nifty because they heavily imply that our classical notions of “local realism” must be incomplete. Namely, that either quantum objects experience instantaneous action-at-a-distance (locality), or they do not have actualized measurement values when they aren’t being actively measured (realism).

This paper is showing a way to produce a visual image of a bell inequality and explores ways that these inequalities can be used for other visual techniques.

[1] https://youtu.be/zcqZHYo7ONs

perl4ever
The point at which I disconnected from understanding physics is not the actual theory, but the stuff people say about locality. I mean, once you have the concept of fields, they're not local, right? So aren't people saying "what is not nonlocal must be local" which doesn't seem like a substantive principle.

Every time I read about the struggle between realism and locality, it makes me feel unhappy because I have a choice between assuming it's me that's really dumb or everyone else. I can't understand why it should be a real problem.

SiempreViernes
Since you have a finite speed for information to propagate, locality can always be defined as "within the lightcone", and so locality is well defined, even for fields.
archgoon
Fields are neither local nor non local. Locality is about how information propagates.

Classical Gravitation defines a force field that is nonlocal, as it propagates instantaneously. Move a mass from point a to point b, and the entire universe immediately updates the force they experience.

Classical Electromagnetism defines a field that only propagates changes at a finite speed. If you move a charged mass from point a to point b, the change in the electric and magnetic fields radiate outward at the speed of light. One way to think about this is to observe that time update equations for E+M only rely on values infinitesimally near a point. If you need to compute the next time step on a grid, you only need to look at the neighbors of a point to figure out how to update a given point.

perl4ever
Ok, but if you think of a field as existing in space at a given time, then essentially all of it is not in your particular spot.

Just saying "this field has a value elsewhere" means you are talking about something that is not near your given point. And having different values at different points is what I thought a field was.

archgoon
Are you saying that electromagnetism is a nonlocal theory because it has a field? That is not the standard meaning of 'nonlocal'.
Jun 11, 2019 · 2 points, 0 comments · submitted by peter_d_sherman
> Are you saying an omniscient being is impossible, because something about quantum mechanics implies that it is impossible to possess all possible knowledge?

> Or are you saying that even if an omniscient being possesses all possible knowledge, they would nevertheless be unable to accurately predict all sequences because of some knowledge we have about quantum mechanics?

That's an extremely astute question. I'm impressed that you managed to realize that that's an important distinction and that you don't know which it is.

It's actually the latter, and incredibly we can prove it. And the proof of this is surprisingly accessible:

https://www.youtube.com/watch?v=zcqZHYo7ONs

jawns
I'm still not sold on this. I'm no quantum mechanics expert, but I do have a philosophy background, and it seems as if true omniscience -- possessing all possible knowledge -- seems like it should transcend this limitation.

Maybe I'm thinking about a form of omniscience that exists outside of time, in which case, of course an omniscient being would know what happens next, because they would know the future just as well as the past. (Example: Any omniscient being will know which photons will pass through a polarizing lens not based on prediction but based on already knowing the outcome.)

thfuran
Such an entity cannot exist.
jawns
Interesting. I can point you to some very influential philosophers who argue that not only can such an entity exist, it necessarily exists.

Can you tell me more about why you think it can't?

gus_massa
Apparently many other influential philosophers disagree, so take that proof with a grain of salt. https://en.wikipedia.org/wiki/Trademark_argument#Criticisms_...
prophesi
Such a statement can't be proven.
thfuran
Sure, just append "unless it turns out we were wrong" to every scientific and mathematical claim if it helps you feel better.
prophesi
I only append that to philosophical claims and string theory.
chrisweekly
"philosophical claims and string theory" sounds redundant to me
gus_massa
There are always hidden assumptions. The proof only applies up to half-omniscience being that know everything in the current and past of the universe, but not in the future.

As a less powerful entity, if you know all the result in the lab until now, you can consider an experiment made 1 hour ago and "predict" the outcome without breaking the laws of quantum mechanics.

Sep 30, 2018 · 3 points, 0 comments · submitted by jstanley
Sep 16, 2017 · 2 points, 0 comments · submitted by ColinWright
Sep 14, 2017 · 3 points, 0 comments · submitted by neic
Complementary video (Bell's Theorem) on MinutePhysics: https://www.youtube.com/watch?v=zcqZHYo7ONs
Sep 13, 2017 · 2 points, 0 comments · submitted by tambourine_man
Sep 13, 2017 · 4 points, 1 comments · submitted by adenadel
vesinisa
Amazing educational video, and really well executed.
Sep 13, 2017 · 2 points, 0 comments · submitted by joaorico
HN Theater is an independent project and is not operated by Y Combinator or any of the video hosting platforms linked to on this site.
~ yaj@
;laksdfhjdhksalkfj more things
yahnd.com ~ Privacy Policy ~
Lorem ipsum dolor sit amet, consectetur adipisicing elit, sed do eiusmod tempor incididunt ut labore et dolore magna aliqua. Ut enim ad minim veniam, quis nostrud exercitation ullamco laboris nisi ut aliquip ex ea commodo consequat. Duis aute irure dolor in reprehenderit in voluptate velit esse cillum dolore eu fugiat nulla pariatur. Excepteur sint occaecat cupidatat non proident, sunt in culpa qui officia deserunt mollit anim id est laborum.